Đến nội dung

Hình ảnh

Tuyển tập các bài toán BDT trong các kì thi năm 2009

- - - - -

  • Please log in to reply
Chủ đề này có 39 trả lời

#21
trungdeptrai

trungdeptrai

    Hạ sĩ

  • Thành viên
  • 53 Bài viết

Đề thi hsg lớp 10
ĐHKHTN(28/8)
Gải sử a,b,c là 3 số thỏa mãn đk $a+b+c=1$
$ \dfrac{a}{1+ a^{2} }+ \dfrac{b}{1+ b^{2} }+ \dfrac{c}{1+ c^{2} } \leq \dfrac{9}{10} $
Bài này mình áp dụng CS dạg engel
Translate into English
Let a,b,c be real numbers such that $a+b+c=1$
Prove that
$ \dfrac{a}{1+ a^{2} }+ \dfrac{b}{1+ b^{2} }+ \dfrac{c}{1+ c^{2} } \leq \dfrac{9}{10} $

hơ,bài này sai rồi thì phải.
Thử: $a=-1,b=0,c=2$
Có lẽ: $a,b,c\geq -\dfrac{3}{4}$

Bài viết đã được chỉnh sửa nội dung bởi trungdeptrai: 12-11-2009 - 03:39


#22
trungdeptrai

trungdeptrai

    Hạ sĩ

  • Thành viên
  • 53 Bài viết

6/ Cho $a,b,c$ là 3 cạnh của tam giác:
CMR:
$\sum \sqrt{a}(\dfrac {1}{b+c-a}-\dfrac {1}{\sqrt{bc}})\geq 0$
ĐHSP TST Dự Tuyến

Vì $a,b,c$ là ba cạnh tam giác nên ta có thể đặt $a=y+z,b=z+x,c=x+y$
Khi đó:
dpcm$\Leftrightarrow \sum_{a,b,c}\sqrt{y+z}\left( \dfrac{1}{2x}-\dfrac{\sqrt{y+z}}{\sqrt{\left( x+y\right)\left( z+x\right)}}\right)\geq 0$
$\Leftrightarrow \sum_{a,b,c}\sqrt{\left( x+y\right)\left( y+z\right)\left( z+x\right)}\sqrt{y+z}\left( \dfrac{1}{2x}-\dfrac{\sqrt{y+z}}{\sqrt{\left( x+y\right)\left( z+x\right)}}\right)\geq 0$
$\Leftrightarrow \sum_{a,b,c}\left( y+z\right)\left( \dfrac{\sqrt{\left( x+y\right)\left( z+x\right)}}{2x}-1\right)\geq 0$$\Leftrightarrow \sum_{a,b,c}\left( y+z\right) \dfrac{\sqrt{\left( x+y\right)\left( z+x\right)}}{2x}\geq 2\left( x+y+z\right)$
Lại có:$\left(x+y \right)\left(x+z \right)\geq {\left( x+\sqrt{yz} \right)}^{2}$(Áp dụng BĐT Cauchy-Schwarz)Tương tự:...
Như vậy ta chỉ cần chứng minh:
$\Leftrightarrow \sum_{a,b,c}\left( y+z\right)\dfrac{x+\sqrt{yz}}{x}\geq 4\left( x+y+z\right)$$\Leftrightarrow \sum_{a,b,c}\left( y+z\right)\left( \dfrac{\sqrt{yz}}{x}+1 \right)\geq 4\left( x+y+z\right)$
$\Leftrightarrow \sum_{a,b,c}\left( y+z\right)\dfrac{\sqrt{yz}}{x}\geq 2\left( x+y+z\right)$
Lại có:$\sqrt{yz}\left( y+z\right)\geq 2yz$(Áp dụng BĐT AM-GM)Tương tự:...
Như vậy ta chỉ cần chứng minh:$\dfrac{xy}{z}+\dfrac{yz}{x}+\dfrac{zx}{y}\geq x+y+z$
BĐT này đúng vì$\dfrac{xy}{z}+\dfrac{yz}{x}\geq 2y$.Tương tự:...cộng lại.
Vậy ta có dpcm...
----------------------------------------------------------------------------------------------------------------------------------------

5/ Với ba số $x,y,z$ dương ta kí hiệu $M$ là số lớn nhất trong ba số
$\ln z+\ln(\dfrac{x}{yz}+1),\ \ln\dfrac{1}{z}+\ln(xyz+1),\ \ln y+\ln(\dfrac{1}{xyz}+1)$
Tìm giá trị nhỏ nhất có thể của $M$ khi $x,y,z$ dương thay đổi.
ĐHSP TST 2009 (Vòng phụ)

Đây là đề kiểm tra đội dự tuyển 2009 ạ,em nghĩ ghi vậy rõ hơn ghi là "vòng phụ".

#23
trungdeptrai

trungdeptrai

    Hạ sĩ

  • Thành viên
  • 53 Bài viết

7/Cho a,b,c là 3 số thực dương thỏa mãn ab+bc+ca=3.Cm rằng
$\[ \dfrac {1}{1+a^{2}(b+c)}+\dfrac {1}{1+b^{2}(c+a)}+\dfrac {1}{1+c^{2}(a+b) }\leq\dfrac {3}{1+2abc}. \]$
MathLinks Contest Edition 7 round 3


Làm nốt bài rồii bố chí đi ngủ thôi...hic...3h ùi...
Bài này mạnh hơn bài Problem 14 trong tuyển tập năm 2008.
$\dfrac {1}{1+a^{2}(b+c)}+\dfrac {1}{1+b^{2}(c+a)}+\dfrac {1}{1+c^{2}(a+b) }\leq\dfrac {3}{1+2abc}$
Áp dụng BĐT Cauchy-Schwarz,ta có:
$\sum_{a,b,c}\dfrac {{a}^{2}\left( b+c\right)}{1+a^{2}(b+c)}\geq \dfrac{{\left( \sum_{a,b,c}a\left( b+c\right)\right)}^{2}}{\sum_{a,b,c}\left( b+c\right)\left(1+a^{2}(b+c)\right)}=\dfrac{36}{ \sum_{a,b,c}\left( b+c\right)\left(1+a^{2}(b+c)\right)}$ (chú ý:$ab+bc+ca=3$)
Ta có:$\sum_{a,b,c}\left( b+c\right)\left(1+a^{2}(b+c)\right)=2\left( a+b+c\right)+\sum_{a,b,c}{\left( ab+ac \right)}^{2}=\sum_{a,b,c}\left( {a}^{2}{b}^{2}+{a}^{2}{c}^{2}+2{a}^{2}bc\right)+2 \left ( a+b+c\right)$
$=2{\left( ab+bc+ca\right)}^{2}+2\left( a+b+c\right)\left( 1-abc\right)=2\left(9+\left( a+b+c\right)\left( 1-abc\right) \right)$
Suy ra ta có:$\sum_{a,b,c}\dfrac {{a}^{2}\left( b+c\right)}{1+a^{2}(b+c)}\geq \dfrac{18}{9+\left( a+b+c\right)\left( 1-abc\right)}$
dpcm$\Leftrightarrow \sum_{a,b,c}\dfrac {{a}^{2}\left( b+c\right)}{1+a^{2}(b+c)}+\dfrac{3}{1+2abc}\geq 3$
Như vậy,ta chỉ cần chứng minh:
$\dfrac{18}{9+\left( a+b+c\right)\left( 1-abc\right)}+\dfrac{3}{1+2abc}\geq 3\Leftrightarrow \dfrac{6}{9+\left( a+b+c\right)\left( 1-abc\right)}+\dfrac{1}{1+2abc}\geq 1(*)$
Đến đây dùng $p,q,r$ có vẻ khá khả thi:D.
$:D\Leftrightarrow \dfrac{6}{9+p\left( 1-r\right)}+\dfrac{1}{1+2r}\geq 1\Leftrightarrow \dfrac{6}{9+p\left( 1-r\right)}\geq \dfrac{2r}{1+2r}$
$\Leftrightarrow 6+12r\geq 18r+2pr-2{p}^{2}{r}^{2}\Leftrightarrow 3\geq \geq 3r+pr-{p}^{2}{r}^{2}\Leftrightarrow \left( r-1\right)\left( pr-3\right)\geq 0$
BĐT trên đúng vì:với mọi $p,q,r$ ta có:
${q}^{2}\geq 3pr\Rightarrow pr\leq 3$
${q}^{3}\geq 27r\Rightarrow r\leq 1$
Vậy ta có dpcm...

Bài viết đã được chỉnh sửa nội dung bởi trungdeptrai: 12-11-2009 - 11:30


#24
vo thanh van

vo thanh van

    Võ Thành Văn

  • Hiệp sỹ
  • 1197 Bài viết
Có thêm 1 số bài sau:
Problem 1:Brazilian Math Olympiad
Let $n > 3$ be a fixed integer and $x_1,x_2,\ldots, x_n$ be positive real numbers. Find, in terms of $n$, all possible real values of
${x_1\over x_n + x_1 + x_2} + {x_2\over x_1 + x_2 + x_3} + {x_3\over x_2 + x_3 + x_4} + \cdots + {x_{n - 1}\over x_{n - 2} + x_{n - 1} + x_n} + {x_n\over x_{n - 1} + x_n + x_1}$
Problem 2:China Girls Math Olympiad
Let $x,y,z$ be real numbers greater than or equal to 1. Prove that
$\prod(x^{2} - 2x + 2)\le (xyz)^{2} - 2xyz + 2.$
Problem 3:Serbian JBTST 4
For positive real numbers $x,y,z$ holds
$\dfrac1{x^2 + 1} + \dfrac1{y^2 + 1} + \dfrac1{z^2 + 1} = \dfrac{1}{2}$
Prove the inequality:
$\dfrac1{x^3 + 2} + \dfrac1{y^3 + 2} + \dfrac1{z^3 + 2} < \dfrac{1}{3}$
Quy ẩn giang hồ

#25
trungdeptrai

trungdeptrai

    Hạ sĩ

  • Thành viên
  • 53 Bài viết

Problem 2:China Girls Math Olympiad
Let $x,y,z$ be real numbers greater than or equal to 1. Prove that
$\prod(x^{2} - 2x + 2)\le (xyz)^{2} - 2xyz + 2.$

Bài này em đã giải bên ddbdt.co.cc
thôi em post luôn ở đây vậy:
Sử dụng bổ đề:
với $a,b\ge 1$thì $\left({a}^{2}-2a+2 \right) \left ({b}^{2}-2b+2 \right) \le { \left (ab \right)}^{2}-2ab+2$
$\leftrightarrow \left(a-1 \right)\left(b-1 \right)\left(a+b-1 \right)\ge 0 $
Suy ra:
$\prod \left({x}^{2}-2x+2\right)\le \left({\left(xy \right)}^{2}-2xy+2 \right)\left({z}^{2}-2z+2 \right)\le {\left(xyz \right)}^{2}-2xyz+2$
Vậy ta có dpcm...
---------------------------------------------------------------------------------------------------------------------------------------------
Còn đây là lời giải của anh Materazzi trên ddbdt.co.cc: http://ddbdt.co.cc/f...wthread.php?t=3

Đặt : $ x=a+1;y=b+1;z=c+1$ ( $a,b,c \ge 0$)

Bất đẳng thức cần chứng minh tương đương :

$2(\sum ab(a+b))+2(ab+bc+ca)+6abc+2abc(ab+bc+ca)+4abc(a+b+c) \ge 0$

Cái này hiển nhiên đúng nên có điều phải chứng minh .


Bài viết đã được chỉnh sửa nội dung bởi trungdeptrai: 12-11-2009 - 16:21


#26
trungdeptrai

trungdeptrai

    Hạ sĩ

  • Thành viên
  • 53 Bài viết

Problem 3:Serbian JBTST 4
For positive real numbers $x,y,z$ holds
$\dfrac1{x^2 + 1} + \dfrac1{y^2 + 1} + \dfrac1{z^2 + 1} = \dfrac{1}{2}$
Prove the inequality:
$\dfrac1{x^3 + 2} + \dfrac1{y^3 + 2} + \dfrac1{z^3 + 2} < \dfrac{1}{3}$


Ta chỉ cần tìm k thỏa mãn các BĐT sau là xong:$\dfrac{1}{{x}^{3}+2}<k \dfrac{1}{{x}^{2}+1}$.Tượng tự:...
Cộng lại:$\sum_{x,y,z}\dfrac{1}{{x}^{3}+2}<k \sum_{x,y,z}\dfrac{1}{{x}^{2}+1}=\dfrac{k}{2}$.Như vậy $k=\dfrac{2}{3}$
Ta chứng minh BĐT đó với $k=\dfrac{2}{3}$:
$\dfrac{1}{{x}^{3}+2}<\dfrac{2}{3\left( {x}^{2}+1\right)}$
$\Leftrightarrow 2{x}^{3}+4 > 3{x}^{2}+3\Leftrightarrow 2{x}^{3}-3{x}^{2}+1>0\Leftrightarrow {\left( x-1\right)}^{2}\left( 2x+1\right)>0$(Đúng do $x=y=z=1$không thỏa mãn ĐK nên dấu "=" không xảy ra)
Vậy ta có dpcm...

Bài viết đã được chỉnh sửa nội dung bởi trungdeptrai: 14-12-2009 - 19:06


#27
trungdeptrai

trungdeptrai

    Hạ sĩ

  • Thành viên
  • 53 Bài viết

8/Cho $a\geq b\geq c\geq d > 0$ thỏa mãn abcd=1.Cm
$\[ \dfrac {1}{1+a}+\dfrac {1}{1+b}+\dfrac {1}{1+c}\geq\dfrac{3}{1+\sqrt[3]{abc}}. \]$
MathLinks Contest Edition 7 round 7

Ơ bài này d cho vào làm gì thế nhỉ???...Anh sửa đề hộ em với.
Mọi người tham gia tích cực vào chứ ạ.:D

#28
nguyen xuan huy

nguyen xuan huy

    Hạ sĩ

  • Thành viên
  • 81 Bài viết
KG biet cua nuoc nao nua
Cho x,y,z>0,$x^2 + y^2 + z^2 = 1$,chứng minh:
$\dfrac{{x + y}}{{1 + xy}} + \dfrac{{y + z}}{{1 + yz}} + \dfrac{{z + x}}{{1 + zx}} \le \dfrac{9}{{2(x + y + z)}}$

#29
nguyen xuan huy

nguyen xuan huy

    Hạ sĩ

  • Thành viên
  • 81 Bài viết
Khoa học tự nhiên 2009

File gửi kèm



#30
vo thanh van

vo thanh van

    Võ Thành Văn

  • Hiệp sỹ
  • 1197 Bài viết
Xin nói rõ với mọi người một chút,đây là Tuyển tập BDT ở các kì thi HSG của các nước trong năm 2009 chứ không phải ở các tỉnh thành,vì thế chúng ta nên tập trung giải các bài BDT của các nước khác thôi ạ.Thank mọi người đã rất hăng hái và năng nổ trong suốt thời gian qua,mong mọi người tiếp tục giữ vững phong độ đó để có thể nhanh chóng hoàn thành tuyển tập.
Thân,
Quy ẩn giang hồ

#31
suguku

suguku

    Trung sĩ

  • Thành viên
  • 126 Bài viết

KG biet cua nuoc nao nua
Cho x,y,z>0,$x^2 + y^2 + z^2 = 1$,chứng minh:
$\dfrac{{x + y}}{{1 + xy}} + \dfrac{{y + z}}{{1 + yz}} + \dfrac{{z + x}}{{1 + zx}} \le \dfrac{9}{{2(x + y + z)}}$

solution
không mất tính tổng quát giả sử $x \geq y \geq z$
ta có $\dfrac{x+y}{1+xy} \geq \dfrac{z+x}{1+zx} \geq \dfrac{y+z}{1+yz}/$ ps:(bdtd)
$1+xy \geq 1+zx \geq 1+yz$
do đó theo bdt TREBUSEP ta có
$( \dfrac{x+y}{1+xy} + \dfrac{z+x}{1+zx} + \dfrac{y+z}{1+yz})(1+xy +1+zx +1+yz)\leq 3(x+y +y+z +z+x)$
VẬY ta chỉ cần cm $ \sum \dfrac{6(x+y+z)}{3+\sum xy} \leq \dfrac{9}{2(x+y+z)} $
bdt này luôn đúng theo cauchy + bdtd
Dau bang xay ra khi $x=y=z= \dfrac{1}{ \sqrt{3} } $
PS:to trungdeptrai anh không biết.đề bài chỉ có vậy thôi.em xem ở đây
http://www.mathlinks...cf81040631bad6a
Sông dài cuồn cuộn ra khơi ,
Anh hùng : sóng dập, cát vùi thiên thu...
Dở hay, thành bại nào đâu?
Bể dâu chớp mắt , nghoảnh đầu thành mơ !
Non xanh còn đó trơ trơ ,
Tà dương lần lửa sưởi hơ ánh hồng.
Lão tiều gặp lại ngư ông ,
Bên sông gió mát , trăng trong , kho trời.
Rượu vò lại rót khuyên mời ,
Cùng nhau lại kể chuyện thời xa xưa...
Kể ra biết mấy cho vừa?
Nói cười hỉ hả , say sưa quên đời...

#32
vo thanh van

vo thanh van

    Võ Thành Văn

  • Hiệp sỹ
  • 1197 Bài viết
Tiếp theo là 2 bài BDT của Kyrgyzstan,have fun ^^
Problem 1:Kyrgyzstan TST 2009
Does $a^2 + b^2 + c^2 \leqslant 2(ab + bc + ca)$ hold for every $a,b,c$ if it is known that $a^4 + b^4 + c^4 \leqslant 2(a^2 b^2 + b^2 c^2 + c^2 a^2 )$
Problem 2:Kyrgyzstan TST 2009
For any positive $a_1 ,a_2 ,...,a_n$ prove that $\dfrac {{a_1 }} {{a_2 + a_3 }} + \dfrac {{a_2 }} {{a_3 + a_4 }} + ... + \dfrac {{a_n }} {{a_1 + a_2 }} > \dfrac {n} {4}$ holds
Quy ẩn giang hồ

#33
Janienguyen

Janienguyen

    Sĩ quan

  • Thành viên
  • 352 Bài viết

Xin nói rõ với mọi người một chút,đây là Tuyển tập BDT ở các kì thi HSG của các nước trong năm 2009 chứ không phải ở các tỉnh thành,vì thế chúng ta nên tập trung giải các bài BDT của các nước khác thôi ạ.Thank mọi người đã rất hăng hái và năng nổ trong suốt thời gian qua,mong mọi người tiếp tục giữ vững phong độ đó để có thể nhanh chóng hoàn thành tuyển tập.
Thân,

Problem (Ian Borsenco,mathematical reflection 2009)
Let a,b,c be real numbers Prove that
$( a^{2} +b^{2} +c^{2})( a^{2}b^{2} +b^{2}c^{2}+c^{2}a^{2}) \geq ( a^{2} + ab+b^{2})(b^{2}+bc +c^{2}) ( a^{2} + ac+c^{2})$
Life is a highway!

#34
trungdeptrai

trungdeptrai

    Hạ sĩ

  • Thành viên
  • 53 Bài viết

Tiếp theo là 2 bài BDT của Kyrgyzstan,have fun ^^
Problem 1:Kyrgyzstan TST 2009
Does $a^2 + b^2 + c^2 \leqslant 2(ab + bc + ca)$ hold for every $a,b,c$ if it is known that $a^4 + b^4 + c^4 \leqslant 2(a^2 b^2 + b^2 c^2 + c^2 a^2 )$
Problem 2:Kyrgyzstan TST 2009
For any positive $a_1 ,a_2 ,...,a_n$ prove that $\dfrac {{a_1 }} {{a_2 + a_3 }} + \dfrac {{a_2 }} {{a_3 + a_4 }} + ... + \dfrac {{a_n }} {{a_1 + a_2 }} > \dfrac {n} {4}$ holds

(Kyrgyzstan TST 2009)
For any positive $a_1 ,a_2 ,...,a_n$prove that $\dfrac {{a_1 }} {{a_2 + a_3 }} + \dfrac {{a_2 }} {{a_3 + a_4 }} + ... + \dfrac {{a_n }} {{a_1 + a_2 }} > \dfrac {n} {4}$ holds
Lời giải của em thế này ạ:
Giả sử $a_i_1$ là số lớn nhất trong các $a_i$.Chọn dãy$a_i_2,a_i_3,...,a_i_k$ với mỗi $a_i_r_+_1$ là số lớn nhất trong 2 số ở mẫu số,còn tử số là $a_i_r$.Như vậy ta có:
$\dfrac{a_1}{a_2+a_3}+\dfrac{a_2}{a_2+a_4}+...+\dfrac{a_n_-_1}{a_n+a_1}+\dfrac{a_n}{a_1+a_2}> \dfrac{a_i_1}{2a_i_2}+\dfrac{a_i_2}{2a_i_3}+...+\dfrac{a_i_k}{2a_i_1}$
$=\dfrac{1}{2}\left( \dfrac{a_i_1}{a_i_2}+\dfrac{a_i_2}{a_i_3}+...+\dfrac{a_i_k}{a_i_1}\right)\geq \dfrac{n}{4}$(Theo BĐT AM-GM)
Vậy ta có dpcm...

Bài viết đã được chỉnh sửa nội dung bởi trungdeptrai: 15-11-2009 - 09:46


#35
vo thanh van

vo thanh van

    Võ Thành Văn

  • Hiệp sỹ
  • 1197 Bài viết

Như vậy ta có:
$\dfrac{a_1}{a_2+a_3}+\dfrac{a_2}{a_2+a_4}+...+\dfrac{a_n_-_1}{a_n+a_1}+\dfrac{a_n}{a_1+a_2}> \dfrac{a_i_1}{2a_i_2}+\dfrac{a_i_2}{2a_i_3}+...+\dfrac{a_i_k}{2a_i_1}$

Em chứng minh rõ đoạn này đi :delta
Quy ẩn giang hồ

#36
abstract

abstract

    Sĩ quan

  • Thành viên
  • 430 Bài viết

hơ,bài này sai rồi thì phải.
Thử: $a=-1,b=0,c=2$
Có lẽ: $a,b,c\geq -\dfrac{3}{4}$

De bai ko sai dau ban ak!
Ban Janienguyen co the post loi giai Cauchy-Swarz dang Engel. Minh to mo muon biet
Đã mang tiếng ở trong trời đất
Phải có danh gì với núi sông


#37
trungdeptrai

trungdeptrai

    Hạ sĩ

  • Thành viên
  • 53 Bài viết

De bai ko sai dau ban ak!
Ban Janienguyen co the post loi giai Cauchy-Swarz dang Engel. Minh to mo muon biet

ah uh,mình nhầm ,rất xin lỗi :icon1:
Bài này cũng có thể giải bằng phương pháp dồn biến ba biến, hay phương pháp dồn theo hai biến bằng nhau :D
Mọi người thử nhé...:Leftrightarrow

Bài viết đã được chỉnh sửa nội dung bởi trungdeptrai: 12-12-2009 - 20:43


#38
vo thanh van

vo thanh van

    Võ Thành Văn

  • Hiệp sỹ
  • 1197 Bài viết
Hiện tại mình đã tập hợp được 20 bài toán đã có lời giải,phần còn lại mình đã gửi cho NguyenDungTN hoàn thành nốt công việc,hi vọng sẽ nhanh chóng hoàn thành.
Quy ẩn giang hồ

#39
T*genie*

T*genie*

    Đường xa nặng bóng ngựa lười...

  • Quản lý Toán Ứng dụng
  • 1161 Bài viết

11/Cho $x_{1},x_{2},\ldots,x_{n}$ là các số lớn hơn 1.Cm rằng
$\[ \dfrac{x_{1}x_{2}}{x_{3}}+\dfrac{x_{2}x_{3}}{x_{4}}+\cdots+\dfrac{x_{n}x_{1}}{x_{2}}\ge4n \]$
Indonesia TST 2009

Nhìn bài sai đề này mình lại nhớ đến một bài đúng đề sau, mọi người thử xem:

Cho số nguyên $n \geq 3 $. Chứng minh rằng với mọi số thực dương $x_{1},x_{2},\ldots,x_{n}$ được sắp xếp theo thứ tự tăng, ta có bđt:

$\dfrac{x_{1}x_{2}}{x_{3}}+\dfrac{x_{2}x_{3}}{x_{4}}+\cdots+\dfrac{x_{n}x_{1}}{x_{2}}\ge x_{1}+x_{2}+\cdots+x_{n}$

Bài viết đã được chỉnh sửa nội dung bởi T*genie*: 13-12-2009 - 00:11


#40
Janienguyen

Janienguyen

    Sĩ quan

  • Thành viên
  • 352 Bài viết

ah uh,mình nhầm ,rất xin lỗi :icon1:
Bài này cũng có thể giải bằng phương pháp dồn biến ba biến, hay phương pháp dồn theo hai biến bằng nhau :D
Mọi người thử nhé...:Leftrightarrow

bài này trong STBDDT cũng có,dùng dồn bien n có đk của a,b,c n k có lời giải
còn cách của mình làm thì có vẻ cũng hơi bất ổn ,tại mình làm từ đợt thi xog mà
hướng của mình là $ \dfrac{(a+1)^2}{1+ a^{2} }+ \dfrac{(b+1)^2}{1+ b^{2} }+ \dfrac{(c+1)^2}{1+ c^{2} } \leq \dfrac{24}{5} $
rồi dự theo dấu= và tách ra
để mình sem xét cụ thể lại rồi sẽ post lg của mình lên sau

Bài viết đã được chỉnh sửa nội dung bởi Janienguyen: 13-12-2009 - 21:12

Life is a highway!




0 người đang xem chủ đề

0 thành viên, 0 khách, 0 thành viên ẩn danh